Which one of the following could be a complete and accurate list of the birds NOT in the forest?

coleagainstcoal on July 31, 2020

Game setup and question explanation

Hello, is there an opportunity to do a game setup and question explanation for this game?

Replies
Create a free account to read and take part in forum discussions.

Already have an account? log in

Victoria on August 1, 2020

Hi @coleagainstcoal,

Happy to help!

This is a group game. We are trying to determine which birds are "in" or "out" of the forest based on the conditions outlined.

We know that there are six possible birds that the forest could contain: G, H, J, M, S, and W.

IN: _ _ _ _ _ _
OUT: _ _ _ _ _ _

Now let's go through the conditions.

Rule 1 - If H are in the forest, then G are not.

H --> Not G
G --> Not H

Rule 2 - If J, M, or both are in the forest, then so are H.

J or M --> H
Not H --> Not J and Not M

Rule 3 - If W are in the forest, then so are G.

W --> G
Not G --> Not W

Rule 4 - If J are not in the forest, then S are.

Not J --> S
Not S --> J

Notice that we can make a transitive chain from the S&N conditions outlined in the rules.

Not S --> J -->
H --> Not G --> Not W
M -->

In this case, J, M, and H would be in and G, W, and S would be out. Remember that we do not need both J and M for H to be present. Therefore, either J or M (but not both) could also be out.

Option 1

IN: J M H
OUT: G W S

Option 2

IN: J H
OUT: M G W S

Option 3

IN: M H S
OUT: J G W

The only thing that changes in option 3 is that S must be in the forest due to Rule 4.

Now let's go through the contrapositive chain:

--> Not J --> S
W --> G --> Not H
--> Not M

In this case, W, G, and S would be in and H, M, and J would be out. We cannot add either J or M here because that would mean that H would be in the forest.

Now that we've got a few different scenarios outlined, let's address the question stem. We are looking for the answer choice which could be a complete and accurate list of birds which are OUT of the forest.

Let's go back through the scenarios we've outlined. Notice that answer choice (D) is reflected in Option 2 outlined above. Therefore, answer choice (D) is correct.

However, let's run through the other answer choices to double check.

Answer choice (A) is incorrect because of Rule 4. If J are out, then S must be in. Therefore, both J and S cannot be out.

Answer choice (B) is incorrect because of Rule 2. We know that if both J and M are in the forest, then so are H. Therefore, this cannot be a complete and accurate list of birds out of the forest because, if H is out of the forest, then both J and M must be out of the forest.

Answer choice (C) is incorrect because of Rule 3. If G are out of the forest, then W must be out of the forest.

Answer choice (E) is incorrect because of Rule 1. If H are in the forest, then G are not.

Hope this is helpful! Please let us know if you have any further questions.

lsatstudier1 on September 15, 2020

FYI - Jelena did an office hours w/ video setup on this one called "Logic Games to Review Part 1"

Mehran on October 8, 2021

This game is also covered in the Group Games lesson. It is example 7.